Find the measure of the indicated angle

Find The Measure Of The Indicated Angle

Answers

Answer 1

Answer:

[tex]given\Longrightarrow AB=AC[/tex]

[tex]\angle \;C=\angle \:B=60\ [isosceles ~triangle][/tex]

[tex]\angle~A+\angle~B+\angle~C=180[/tex]

[tex]x+60+60=180[/tex]

[tex]x+120=180[/tex]

[tex]x=180-120[/tex]

[tex]x=60[/tex]

-------------------------

HOPE IT HELPS

HAVE A GREAT DAY!!


Related Questions

The center of the circle is at the point
, and its radius is
units. The equation of this circle in standard form is
.

Answers

Is there a picture that goes with this?

Choose the smallest number 3 1/8 or 10/3

Answers

Answer:

10/3

Step-by-step explanation:

31/8= 3.8

10/3= 3.3

What’s the equation of the line that passes through the point (-4,4) and has a slope of 3/4

Answers

Answer:

y-y1=m(x-x1)

y-4=3/4(x+4)

y=3/4x+7

Evaluate 2y when y = 6y

Answers

I believe the answer is 12y
im sure the answer is 12y

A statistics professor asked students in a class their ages. On the basis of this information, the professor states that the average age of all the students in the university is 24 years. This is an example of

Answers

Answer:

propbability ???

Step-bp explanation:

Answer:

Step-by-step explanation:

This is an example of a statistical mean.

What is the quotient of (x^3 - 3x^2 + 3x - 2) ÷ (x^2 - x + 1)?
O x - 2
O x + 2
O x- 4
O x + 1

Answers

Answer:

x-2

The choose (1)

Step-by-step explanation:

(x³-3x²+3x-2)÷(x²-x+1)

(x-2)(x²-x+1) ÷ (x²-x+1)

Delete (x²-x+1)

so = (x-2)

find the domain of f(x)=sec(2x)

Answers

Answer:

*Refer the image attached

Step-by-step explanation:

*Refer the image attached

someone help me for this algebra task please

Answers

i just need to answer questions

According to the number line, what is the distance between points A and B?

0 6 units
7 units
O 12 units
O 14 units

Answers

Answer:

14 units

Step-by-step explanation:

A = - 2, B = 12

Therefore,

d(A, B) = 12 - (-2) = 12 + 2 = 14 units

solve for x. Round to the nearest tenth, if necessary.​

Answers

Answer:

7.1

Step-by-step explanation:

We used SOHCAHTOA because it's a right angle triangle

So because we have an angle with an adjacent of 6.3 and hypotenuse of x

We will use

Cos=adjacent /hypotenuse

7.1 like the last answer if not you can round that

There are 35 times as many students at Wow University as teachers. When all the students and
teachers are seated in the 8544 seat auditorium, 12 seats are empty. How many students attend
Wow University?

Answers

Given:

There are 35 times as many students at Wow University as teachers.

When all the students and teachers are seated in the 8544 seat auditorium, 12 seats are empty.

To find:

The total number of students.

Solution:

Let x be the number of teachers at Wow University. So, the number of student is :

[tex]35\times x=35x[/tex]

When all the students and teachers are seated in the 8544 seat auditorium, 12 seats are empty.

[tex]x+35x=8544-12[/tex]

[tex]36x=8532[/tex]

[tex]x=\dfrac{8532}{36}[/tex]

[tex]x=237[/tex]

The number of total students is:

[tex]35x=35(237)[/tex]

[tex]35x=8295[/tex]

Therefore, the total number of students is 8295.

jordan wants to save to buy a car and decides to open a banking account that is offering an interest rate of 4.5% compounded annually how much will jordan have in the account after 5 years it he deposits $7,000 today?

Answers

Answer:

8,723.27$

Step-by-step explanation:

On the unit circle, which of the following angles has the terminal point
coordinates.
A. 45
B. 135
C. 225
D. 315

Answers

Answer: C. 225

Step-by-step explanation:

The product of integers a,b,c and d is 546 and if 1<a<b<c<d, what is the value of b+c?​

Answers

Hello,

546=2*3*7*13

a=2, b=3,c=7,d=13

b+c=3+7=10

pls help me don't know what to do

Answers

Answer:

x=15

Step-by-step explanation:

The 60 degree angle and the (x+45) degree angle are both the same degree because they are vertical angles.

So to solve, just subtract 45 from 60

60-45=15

That's your answer!

Hope this helps!

Select the expression that represents the following statement: add 24 to the quotient of 16 and 8.

Answers

Answer:

16/8 + 24

Step-by-step explanation:

find the slope of the line passing through the points (-2,5) and (3/2,2)

Answers

Answer:

slope = - [tex]\frac{6}{7}[/tex]

Step-by-step explanation:

Calculate the slope m using the slope formula

m = [tex]\frac{y_{2}-y_{1} }{x_{2}-x_{1} }[/tex]

with (x₁, y₁ ) = (- 2, 5) and (x₂, y₂ ) = ([tex]\frac{3}{2}[/tex], 2)

m = [tex]\frac{2-5}{\frac{3}{2}-(-2) }[/tex]

   = [tex]\frac{-3}{\frac{3}{2}+2 }[/tex]

   = [tex]\frac{-3}{\frac{7}{2} }[/tex]

  = - 3  × [tex]\frac{2}{7}[/tex]

  = - [tex]\frac{6}{7}[/tex]

The cost for 2 adults and 5 children to eat at the local buffet restaurant is \$46.50 . The cost for 1 adult and 1 child is $15. What is the cost of a child's meal ?

Answers

Answer:

c = 5.5

Step-by-step explanation:

2 a + 5 c = 46.50

a + c = 15

a = 15 - c

2(15-c ) +5c = 46.50

30 - 2c +5c = 46.5

3c = 16.5

c = 5.5

Let a be the adults and c be the children.

For the first equation there are 2 adults and 5 children so the equation is:-

2a + 5c = 46.50

And for the second equation there is 1 adult and 1 child. So the equation is:-

a + c = 15

a = 15 - c

You take this formula and substitute it to the above formula (2a + 5c = 46.50)

➡️ 2a + 5c = 46.50

➡️ 2(15 - c) + 5c = 46.50

➡️ 30 -2c + 5c = 46.50

➡️ 5c - 2c = 46.50 - 30
(You collect like terms)

➡️ 3c = 16.5

➡️ c = 16.5/3

➡️ c = 5.5

2 1/4 x 3 1/5 brainliest

Answers

Answer:

36/5

Step-by-step explanation:

9/4×16/5

144/20

36/5

hope this is helpful

Answer:

[tex]7\frac{1}{5}[/tex]

Step-by-step explanation:

1. start by turning the fractions improper fractions:

[tex]2\frac{1}{4} =\frac{9}{4}[/tex]

[tex]3\frac{1}{5} =\frac{16}{5}[/tex]

2. then multiply them together:

[tex]\frac{9}{4}[/tex] x [tex]\frac{16}{5}[/tex] = [tex]\frac{144}{20}[/tex]

3. then simplify the fraction:

[tex]\frac{144}{20}[/tex][tex]=\frac{36}{5}[/tex]

4. turn it into a proper fraction:

[tex]\frac{36}{5} =7\frac{1}{5}[/tex]

Hello, Brainly community!

This question is for all of those Calculus people out there.

The volume of a swimming pool is changing with respect to time, such that the volume is given by W(t), where W(t) is measured in cubic centimeters and t is measured in seconds. A tangent line is shown for W(t) at t = 3 seconds. Determine the best estimate for the value of the instantaneous rate of change of W(t) when t = 3.
(I've narrowed down the answer choices to 2, and just really need to find the right way of thinking to find the answer)

(A) W(lim t) as t goes to 3.
(B) [W(3.1) - W(2.9)] / 0.2.

Thank you in advance!

Answers

Answer:

(B)  [tex]\displaystyle \frac{W(3.1) - W(2.9)}{0.2}[/tex]

General Formulas and Concepts:

Calculus

Limits

Derivatives

The definition of a derivative is the slope of the tangent line.

Derivative Notation

Instantaneous Rates

Tangent Line: [tex]\displaystyle f'(x) = \frac{f(b) - f(a)}{b - a}[/tex]

Step-by-step explanation:

Since we are trying to find a rate at which W(t) changes, we must find the derivative at t = 3.

We are given 2 close answer choices that would have the same numerical answer but different meanings:

(A)  [tex]\displaystyle \lim_{t \to 3} W(t)[/tex](B)  [tex]\displaystyle \frac{W(3.1) - W(2.9)}{0.2}[/tex]

If we look at answer choice (A), we see that our units would simply just be volume. It would not have the units of a rate of change. Yes, it may be the closest numerically correct answer, but it does not tell us the rate at which the volume would be changing and it is not a derivative.

If we look at answer choice (B), we see that our units would be cm³/s, and that is most certainly a rate of change. Answer choice (B) is also a derivative at t = 3, and a derivative tells us what rate something is changing.

∴ Answer choice (B) will give us the best estimate for the value of the instantaneous rate of change of W(t) when t = 3.

Topic: AP Calculus AB/BC (Calculus I/I + II)

Unit: Differentiation

Book: College Calculus 10e

Explain how the quotient of powers was used to simplify this expression

Answers

Answer:

See explanation

Step-by-step explanation:

Given:

5⁴/25 = 5²

25 = 5²

Then,

5⁴ / 25

= 5⁴/5²

Note:

• multiplication sign means addition in indices

• Division sign means subtraction in indices

Both numerator and denominator have the same base, so you'll pick one of the bases and subtract the powers

So,

5⁴/5²

= 5^(4 - 2)

= 5^(2)

= 5²

Therefore,

5⁴ / 25 = 5²

lidentify the domain of the function shown in the graph
O A 15257
O B. 19334
O C. 221
O D. All real numbers

Answers

Answer:

B.

Step-by-step explanation:

the visible line is the defined function.

this line goes from x=1 to x=4, and has the functional results from y=1 to y=7.

the domain is the valid interval of the input variable (typically x), while the range is the valid inescapable of the result variable (typically y).

so, B is the right answer.

Which operation will solve the following word problem? Jeff earns $14.00 per hour, Tom earns half as much as Jeff. How much does Tom earn per hour?


Multiplication


Subtraction


Addition


Division

Answers

Answer:

The correct option is (d).

Step-by-step explanation:

Given that,

Jeff earns $14.00 per hour.

Tom earns half as much as Jeff.

We need to find the amount earn by Tom per hour.

Tom's amount = Jeff's amount/2

So,

[tex]T=\dfrac{14}{2}\\\\T=\$7[/tex]

So, Tom earn $7 per hour. Hence, division operation is used. Jeff's amount is divided by 2.

The area of the rectangular sandbox at Dave's school is 117 square feet. The sandbox has a width of 9 feet as shown in the diagram. What is the perimeter of the sandbox?

Answers

Answer:

ay bru ima tell yu dhis rn is c

Step-by-step explanation:

Greg buys 60 garden plants at a cost price of $2.00 each to sell in his shop. He sells 25 of them at the profit of 75% and 18 of them at the profit of 35%. He sells the rest of the plants for 4/5 of the cost price calculate the profit or loss he makes from selling 60 plants stating if it is a profit or loss

Answers

Answer:

$43.30 profit

Step-by-step explanation:

Total cost of plant:

60*2 = 120

Greg makes total of:

25*(2 + 0.75*2) + 18*(2 + 0.35*2) + (60 - 25 - 18)*2*4/5 = 163.3

Since Greg mare than cost, he has a profit and the amount is:

163.3 - 120 = 43.3

PLS HELP SOON WILL MARK BRAINLYEST

A railroad tunnel is shaped like a semi-ellipse, as shown below. A semiellipse is shown on the coordinate plane with vertices on the x axis and one point of intersection with the positive y axis. The height of the tunnel at the center is 35 ft, and the vertical clearance must be 21 ft at a point 8 ft from the center. Find an equation for the ellipse.

Answers

According to the question

b= 35 and (8,21) lies on the ellipse

After calculation we get a= 10

equation for the ellipse.

[tex] \frac{ {x}^{2} }{100} + \frac{ {y}^{2} }{1225} = 1[/tex]

If a bus travel for 120 minutes at a speed of 75 kilometers per hour how far has the bus traveled?

Answers

Answer:

150 km

Step-by-step explanation:

Put the minutes into hours 120min is 2 hours.

Distance = speed * time

Distance = 75 * 2

Distance = 150

Answer:

150 kilometers

Step-by-step explanation:

if the bus is going 75 kilometers an hour and they traveled for 120 minutes (exactly 2 hours) then you would just multiply 75 by 2 to get 150 kilometers total.

What type of health screening would this patient most likely receive?
Sue is a 45-year-old woman with a family history of breast cancer. Her healthcare professional will most likely recommend that she receive a .

Answers

Answer:

A mammogram is what she would receive

Step-by-step explanation:

A cinema is doing a promotion to celebrate their 50th anniversary for 1 week. They give

away a free drink to every 98th customer, a free bag of popcorn to every 112th customer and

a free cinema ticket to every 224th customer. Which lucky customer will be the first to

receive all 3 items?​

Answers

Answer:

1,568 customer

Step-by-step explanation:

Find the lowest common multiple of 98, 112, and 224

98 = 98, 196, 294, 392, 490, 588, 686, 784, 882, 980, 1078, 1176, 1274, 1372, 1470, 1568, 1666

112 = 112, 224, 336, 448, 560, 672, 784, 896, 1008, 1120, 1232, 1344, 1456, 1568, 1680, 1792, 1904

224 = 224, 448, 672, 896, 1120, 1344, 1568, 1792, 2016, 2240

The lowest common multiple of 98, 112, and 224 is 1568

Therefore, the 1,568th customer will be the first to receive all 3 iitem

The formula for the circumference of a circle is R = c/2(pi)

Find the radius of a circle that has a circumference of 16(pi)

A) r = 4
B) r = 8
C) r = 12
D) r = 16

Answers

The answer is B) r= 8
Other Questions
Find the measure of the missing angle using the exterior angle sum theorm CAN SOMEONE HELP ME AND EXPLAIN ASAP THANKS Lakeview Engine, Inc., produces engines for the watercraft industry. An outside manufacturer has offered to supply several component parts used in the engine assemblies, which are currently being produced by Lakeview. The supplier will charge Lakeview $325 per engine for the set of parts. Lakeviews current costs for those part sets are direct materials, $145; direct labor, $85; and manufacturing overhead applied at 100% of direct labor. Variable manufacturing overhead is considered to be 20% of the total, and fixed overhead will not change if the part sets are acquired from the outside supplier.What would be the net cost advantage or disadvantage if Lakeview decided to purchase the parts? El Nio is a well- known weather phenomenon. In Ecuador and Peru, fisherman knew about the phenomena long before it was well known because their fishing would be very poor during El Nio conditions. Normally Peru and Ecuador have a remarkable fishery because there is a prominent upwelling along the coast producing planktonic blooms that feed the entire ecosystem. This upwelling is driven by surface wind conditions. From your knowledge of currents and wind conditions, which of the following happens during El Nio conditions in this area? A) The winds change from east to west offshore flow) to onshore flow or weak winds during El Nio conditions. B) The winds shift from prevailing southwesterlies to stagnant, doldrum conditions during El Nio. C) The winds shift from west to east onshore flow) to offshore flow east to west) during El Nio conditions. D) The winds shift from south to north, along the coast, to west to east, across the coast. The Best Company is reviewing two options for replacing a piece of machinery. The first machine costs $100,230 and has a four-year life. The second machine costs $155,000 and has a six-year life. Neither machine will have a salvage value. The machines will be replaced at the end of their life. What method should be used to determine which machine to purchase? A process needs 103 KB of memory in order to run. If the system on which it is to run uses paging with 2 KB pages, how many frames in memory are needed "Cory is a salesperson at Bayray Technologies, a company that manufactures laptops and cameras. During a sales presentation for the company's new camera, he states, "Cameras produced by us have the same resolution as the cameras produced by ClarityO, the world's number one camera manufacturer, but at a cheaper price." In this scenario, Cory is using a(n) _____ to describe the product." Jalen's Mobile Phone Cost Number of minutes, x 150 220 250 275 Cost, y $7.50 $11.00 $12.50 $13.75 If the cost varies directly with the number of minutes Jalen talks on the phone, which equation represents the variation? que contiene el condn? Convert 1.5% to decimal and a fraction. Show and explain your method. Forms of inheritance that do not follow typical Mendelian patterns and that appear to be more influenced by the parent contributing the most cytoplasm to the embryo are grouped under the general heading of ________. b) He was an angel of peace in interrogative sentence what single transformation maps triangle ABC onto ABC? In the xy-plane, the slope of the line y = mx 4 isless than the slope of the line y = x 4. Which of thefollowing must be true about m? [Show Workings}I will give brainlist to the person with the right Which equation represents a line which is parallel to the line y=3x8 help please i don't know how to do this For this question, you will assume the role of two acquaintances living in different countries at the end of the 19th century. First, write a letter from a British citizen to an acquaintance in Japan, China, or Russia describing the industrialization of Great Britain over the last 150 years. Then write a reply from the Japanese, Chinese, or Russian citizen explaining how industrialization in that country has been different. Be sure to acknowledge both positive and negative aspects of industrialization, and support all claims with historical evidence. Michael believes that a piece of equipment he bought will quickly sculpt his abdominal muscles. What can this belief be best identified as?A fitness fad?A safety plan?A workout routine?An equipment manual? como se llama el arte de pintar en las cuevas sus actividades de caceria Read the passage below from "Marigolds" and answer the question.I had Indeed lost my mind, for all the smoldering emotions of that summer swelled In me and burst-the greatneed for my mother who was never there, the hopelessness of our poverty and degradation, thebewilderment of being neither child nor woman and yet both at once, the fear unleashed by my father's tears.And these feelings combined in one great Impulse toward destruction.Based on the passage above, which of the following themes are evident in the story?loss of innocencegood overcomes evilvalue of familylove and sacrifice